Đến nội dung

Hình ảnh

Topic tổng hợp một số bất đẳng thức trong kì thi MO các nước

* * * * * 16 Bình chọn

  • Please log in to reply
Chủ đề này có 501 trả lời

#361
hoctrocuaHolmes

hoctrocuaHolmes

    Thượng úy

  • Thành viên
  • 1013 Bài viết

 Cách khác cho í sau 

 Đặt $a=\frac{1}{3}+x~;b=\frac{1}{3}+y$ với $x,y\geq \frac{-1}{3}$

       $\Rightarrow c=\frac{1}{3}-x-y$ 

       $\Rightarrow ab+bc+ca-2abc=2xy(x+y)-\frac{x^2+y^2+xy}{3}+\frac{7}{27}$

        $=\frac{1}{12}.(6y-1)(2x+y)^2-\frac{1}{4}.y^2(2y+1)+\frac{7}{27}$

 Giả sử $x\geq y\Rightarrow \frac{1}{3}> x+y\geq 2y$

            $\Rightarrow y< \frac{1}{6}$

            $\Rightarrow \frac{1}{12}.(6y-1)(2x+y)^2-\frac{1}{4}.y^2(2y+1)\leq 0$

            $\Rightarrow ab+bc+ca-2abc\leq \frac{7}{27}$

Còn có cách này nữa ạ  :)

Chứng minh vế thứ nhất:Ta có $ab+bc+ca-2abc\geq 0\Leftrightarrow (a+b+c)(ab+bc+ac)-2abc\geq 0\Leftrightarrow a^{2}b+a^{2}c+b^{2}c+b^{2}a+c^{2}a+c^{2}b+abc\geq 0$ (luôn đúng)

Vậy bất đẳng thức đã được cm

Chứng minh vế thứ hai: Ta có bất đẳng thức phụ $(a+b+c)^{3}+9abc\geq 4(a+b+c)(ab+bc+ca)$ (hệ quả của bất đẳng thức Schur )

Áp dụng vào bài ta suy ra $9abc+1\geq 4(ab+bc+ca)\Leftrightarrow \frac{18abc+2}{9}\geq \frac{8(ab+bc+ca)}{9}\Leftrightarrow 2abc+\frac{2}{9}\geq \frac{8}{9}(ab+bc+ca)\Leftrightarrow ab+bc+ca-2abc\leq ab+bc+ca-\frac{8}{9}(ab+bc+ca)+\frac{2}{9}=\frac{1}{9}(ab+bc+ca)+\frac{2}{9}$

Lại có $ab+bc+ca\leq \frac{(a+b+c)^{2}}{3}=\frac{1}{3}\Rightarrow \frac{1}{9}(ab+bc+ca)+\frac{2}{9}\leq \frac{1}{9}.\frac{1}{3}+\frac{2}{9}=\frac{7}{27}$ (đpcm)



#362
Bui Ba Anh

Bui Ba Anh

    Thiếu úy

  • Thành viên
  • 562 Bài viết

Bài 132:(IMO 1984) Cho $a,b,c>0$ thoả mãn $a+b+c=1$.Chứng minh rằng $0\leq ab+bc+ca-2abc\leq \frac{7}{27}$

Bài này mình nghĩ dùng $p,q,r$ là nhanh nhất, tư tưởng có vẻ hợp lí

CM vế đầu: Ta có $ab+ac+bc-2abc=(\sum ab)(\sum a)-2abc \geq 7abc \geq 0$

CM vế sau: Đặt $p=\sum a, q=\sum ab, r=abc$, theo $schur$ ta có : $p^3+9r \geq 4pq=>r\geq \frac{4q-1}{9}$

Ta quy về chứng minh $q-2\frac{4q-1}{9}\leq \frac{7}{27}<=>q \leq \frac{1}{3}$ (đpcm)

Spoiler


Bài viết đã được chỉnh sửa nội dung bởi Bui Ba Anh: 20-06-2015 - 23:00

NgọaLong

#363
dogsteven

dogsteven

    Đại úy

  • Thành viên
  • 1567 Bài viết

Bài này thực sự là đặc chưng cho phương pháp tiếp tuyến, ta giải như sau:

1) Áp dụng bất đẳng thức AM-GM và giả thuyết thì chúng ta sẽ có BĐT đã cho tương đương BĐT sau:

$\sum \frac{1}{a^2-6a+13}\leq \frac{3}{8}$

Thật vậy lưu ý bất đẳng thức phụ sau:

$\frac{1}{a^2-6a+13}\leq \frac{a+1}{16}$

tương tự như thế suy ra$\sum \frac{1}{a^2-6a+13}\leq \frac{a+b+c+3}{16}=\frac{3}{8}$

từ đó ta có đpcm

Giả sử $a\geqslant b\geqslant c$. Bất đẳng thức trên tương đương với: $\sum \dfrac{(a+b)^2}{(a+b)^2+\dfrac{2(a+b)^2}{a^2+b^2}}\geqslant \dfrac{3}{2}$

Áp dụng bất đẳng thức Cauchy-Schwarz: $VT\geqslant \sum \dfrac{4(a+b+c)^2}{\sum (a+b)^2+\sum \dfrac{2(a+b)^2}{a^2+b^2}}$

Do đó ta cần chứng minh: $\sum (a+b)^2+\sum \dfrac{2(a+b)^2}{a^2+b^2}\leqslant 24$

$\Leftrightarrow \dfrac{4}{3} (a+b+c)^2-\sum (a+b)^2+12-\sum \dfrac{2(a+b)^2}{a^2+b^2}\geqslant 0\Leftrightarrow \sum (a-b)^2\left(\dfrac{6}{a^2+b^2}-1\right)\geqslant 0$

Nếu $a^2+b^2\leqslant 6$ thì bất đẳng thức đúng. Nếu $a^2+b^2\geqslant  6$ thì:

$\sum \dfrac{1}{a^2+b^2+2}\leqslant \dfrac{1}{8}+\dfrac{1}{a^2+2}+\dfrac{1}{b^2+6} \leqslant \dfrac{1}{8}+\dfrac{1}{8-b^2}+\dfrac{1}{b^2+2}\leqslant \dfrac{1}{8}+\dfrac{5}{8}=\dfrac{3}{4}$


Bài viết đã được chỉnh sửa nội dung bởi dogsteven: 21-06-2015 - 07:09

Quyết tâm off dài dài cày hình, số, tổ, rời rạc.


#364
dangkhuong

dangkhuong

    Sĩ quan

  • Thành viên
  • 312 Bài viết

Bài 134:(Baltic Way Shortlist) :Cho $a,b,c>0: a+b+c=1$. CMR:

$\frac{a^3+bc^2+ca^2}{b(c+2a)}+\frac{b^3+ca^2+ab^2}{c(a+2b)}+\frac{c^3+ab^2+bc^2}{a(b+2c)}\geq \frac{216abc+1}{9}$


Bài viết đã được chỉnh sửa nội dung bởi ducvipdh12: 21-06-2015 - 19:55

:ukliam2:  :ukliam2:  :ukliam2:


#365
dangkhuong

dangkhuong

    Sĩ quan

  • Thành viên
  • 312 Bài viết

Bài135(Mongolia MO): Cho $a,b,c$ là các số thực không âm. CMR:

$\sum \frac{a^2}{b^2-bc+c^2}\geq 2$


Bài viết đã được chỉnh sửa nội dung bởi ducvipdh12: 21-06-2015 - 19:56

:ukliam2:  :ukliam2:  :ukliam2:


#366
canhhoang30011999

canhhoang30011999

    Thiếu úy

  • Thành viên
  • 634 Bài viết

Bài135(Mongolia MO): Cho $a,b,c$ là các số thực không âm. CMR:

$\sum \frac{a^2}{b^2-bc+c^2}\geq 2$

giả sử c= Min {a,b,c}

ta có $VT= \frac{c^{2}}{a^{2}-ab+b^{2}}+\frac{a^{2}}{b^{2}+c(c-b)}+\frac{b^{2}}{a^{2}+c(c-a)}$

$\geq \frac{a^{2}}{b^{2}}+\frac{b^{2}}{a^{2}}\geq 2$



#367
olympiachapcanhuocmo

olympiachapcanhuocmo

    Thượng sĩ

  • Thành viên
  • 208 Bài viết

Bạn  có thể giải thích chi tiết được không ?


                                                                                               


#368
canhhoang30011999

canhhoang30011999

    Thiếu úy

  • Thành viên
  • 634 Bài viết

Bạn  có thể giải thích chi tiết được không ?

thì $\frac{c^{2}}{a^{2}-ab+b^{2}}\geq 0$

$\frac{b^{2}}{a^{2}-ac+c^{2}}\geq \frac{b^{2}}{a^{2}}$

$\frac{a^{2}}{b^{2}-bc+c^{2}}\geq \frac{a^{2}}{b^{2}}$



#369
olympiachapcanhuocmo

olympiachapcanhuocmo

    Thượng sĩ

  • Thành viên
  • 208 Bài viết

Tớ làm cách này không biết có đúng không  : $\sum \frac{a^{2}}{b^{2}-bc+c^{2}}\geq \sum \frac{a^{2}}{\sqrt{\left ( 2b^{2} +c^{2}\right )\left ( 2c^{2}+b^{2} \right )}}\geq \sum \frac{a^{2}}{b^{2}+c^{2}}.\frac{2}{3}\geq \frac{3}{2}.\frac{2}{3}=1$ ?


                                                                                               


#370
an1712

an1712

    Trung sĩ

  • Thành viên
  • 149 Bài viết

Bài 134:(Baltic Way Shortlist) :Cho $a,b,c>0: a+b+c=1$. CMR:

$\frac{a^3+bc^2+ca^2}{b(c+2a)}+\frac{b^3+ca^2+ab^2}{c(a+2b)}+\frac{c^3+ab^2+bc^2}{a(b+2c)}\geq \frac{216abc+1}{9}$

ta có :

$\sqrt[3]{3b(2c+a)(2a+c)}\leq a+b+c$

$\Leftrightarrow \sum \frac{a^3+bc^2+ca^2}{b(c+2a)}\geq \sum \frac{3(2c+a)(a^3+bc^2+ca^2)}{(a+b+c)^3}$

$bđt \Leftrightarrow 3((a^2+b^2+c^2)^2+5(ab^3+bc^3+ca^3)+abc)\geq \frac{216abc+1}{9}$

áp dụng holder:

$(ab^3+bc^3+ca^3)(\sum \frac{1}{a})(1+1+1)\geq (a+b+c)^3=1$ $\Leftrightarrow 5(\sum ab^3)\geq \frac{5abc}{ab+bc+ac}$

đặt : $a+b+c=p=1$ $ab+ac+bc=q\leq \frac{1}{3}$ $abc=r$

$bdt\Leftrightarrow (p^2-2q)^2+\frac{5r}{3q}-7r-\frac{1}{27}\geq 0\Leftrightarrow (1-2q)^2+\frac{5r}{3q}-7r-\frac{1}{27}$

xét hàm :

TH1

$\frac{5}{21}\geq q$ hàm f'(r) đồng biến

theo shur :$r\geq \frac{4q-1}{9}$

$\Rightarrow f(r)\geq f(\frac{4q-1}{9})\geq 0$ (luôn đúng)

TH2

$\frac{5}{21}\leq q\leq \frac{1}{3}$ (nghịch biến)

có:$\prod (a-b)^2\geq 0\Leftrightarrow \frac{(1+2\sqrt{1-3q})(1-\sqrt{1-3q})^2}{27}\geq r$

$f(r)\geq f(\frac{(1+2\sqrt{1-3q})(1-\sqrt{1-3q})^2}{27})\geq 0$(luôn đúng)

=> đpcm


Bài viết đã được chỉnh sửa nội dung bởi an1712: 21-06-2015 - 22:12

tiến tới thành công  :D


#371
dangkhuong

dangkhuong

    Sĩ quan

  • Thành viên
  • 312 Bài viết

ta có :

$\sqrt[3]{3b(2c+a)(2a+c)}\leq a+b+c$

$\Leftrightarrow \sum \frac{a^3+bc^2+ca^2}{b(c+2a)}\geq \sum \frac{3(2c+a)(a^3+bc^2+ca^2)}{(a+b+c)^3}$

$bđt \Leftrightarrow 3((a^2+b^2+c^2)^2+5(ab^3+bc^3+ca^3)+abc)\geq \frac{216abc+1}{9}$

áp dụng holder:

$(ab^3+bc^3+ca^3)(\sum \frac{1}{a})(1+1+1)\geq (a+b+c)^3=1$ $\Leftrightarrow 5(\sum ab^3)\geq \frac{5abc}{ab+bc+ac}$

đặt : $a+b+c=p=1$ $ab+ac+bc=q\leq \frac{1}{3}$ $abc=r$

$bdt\Leftrightarrow (p^2-2q)^2+\frac{5r}{3q}-7r-\frac{1}{27}\geq 0\Leftrightarrow (1-2q)^2+\frac{5r}{3q}-7r-\frac{1}{27}$

xét hàm :

TH1

$\frac{5}{21}\geq q$ hàm f'(r) đồng biến

theo shur :$r\geq \frac{4q-1}{9}$

$\Rightarrow f(r)\geq f(\frac{4q-1}{9})\geq 0$ (luôn đúng)

TH2

$\frac{5}{21}\leq q\leq \frac{1}{3}$ (nghịch biến)

có:$\prod (a-b)^2\geq 0\Leftrightarrow \frac{(1+2\sqrt{1-3q})(1-\sqrt{1-3q})^2}{27}\geq r$

$f(r)\geq f(\frac{(1+2\sqrt{1-3q})(1-\sqrt{1-3q})^2}{27})\geq 0$(luôn đúng)

=> đpcm

Thực ra cách làm của bạn khá phức tạp sau đây là lời giải của mình:

 

 

Áp dụng bất đẳng thức Cauchy-Schawz ta có:

$\sum \frac{a^3+bc^2+ca^2}{b(c+2a)}\geq \frac{(\sum \sqrt{a^3+bc^2+ca^2})^2}{3(ab+ca+bc)}$

Ta đi cm: $\sum \sqrt{a^3+bc^2+ca^2}\geq 1$

Thật vậy theo bất đẳng thức Cauchy-Schawz thì ta có:$(a^3+bc^2+ca^2)(a+b+c)\geq (a^2+bc+ca)^2$. Do đó ta có$\sqrt{a^3+bc^2+ca^2}\geq a^2+bc+ca$

Từ đó ta sẽ có $\sum \sqrt{a^3+bc^2+ca^2}\geq (a^2+b^2+c^2+2ab+2bc+2ca)=(a+b+c)^2=1$

Do AM-GM và giả thuyết hiển nhiên 3(ab+bc+ca)$\leq$1

Vậy VT bất đẳng thức đã cho$\geq$1=$(a+b+c)^3=a^3+b^3+c^3+3(a+b)(b+c)(c+a)\geq \frac{(a+b+c)^3}{9}+24abc\geq \frac{126abc+1}{9}$ (đpcm).


Bài viết đã được chỉnh sửa nội dung bởi dangkhuong: 22-06-2015 - 15:32

:ukliam2:  :ukliam2:  :ukliam2:


#372
dangkhuong

dangkhuong

    Sĩ quan

  • Thành viên
  • 312 Bài viết

Bài 136(IMO Shortlist): Cho a,b,c là các số thực không âm: ab+bc+ca=1. CMR:

$2\leq \frac{1}{2a^2+bc}+\frac{1}{2b^2+ca}+\frac{1}{2c^2+ab}\leq 3(a^2+b^2+c^2)$


:ukliam2:  :ukliam2:  :ukliam2:


#373
dangkhuong

dangkhuong

    Sĩ quan

  • Thành viên
  • 312 Bài viết

Bài 137(Brirstish MO): Cho a,b,c>0. CMR: nếu a+b+c=3.

$\frac{a}{b^2+8bc+3c^2}+\frac{b}{c^2+8ca+3a^2}+\frac{c}{a^2+8ab+3b^2}\geq \frac{1}{4}$


:ukliam2:  :ukliam2:  :ukliam2:


#374
dangkhuong

dangkhuong

    Sĩ quan

  • Thành viên
  • 312 Bài viết

Bài 138(tự sáng tác)(1 bài tập chặt hơn bài 136):  Cho a,b,c >0:ab+ca+bc=1. CMR:

$\frac{1}{a^2+b^2+bc}+\frac{1}{b^2+c^2+ca}+\frac{1}{c^2+a^2+ab}\leq 3(a^2+b^2+c^2)$


Bài viết đã được chỉnh sửa nội dung bởi dangkhuong: 22-06-2015 - 15:58

:ukliam2:  :ukliam2:  :ukliam2:


#375
dangkhuong

dangkhuong

    Sĩ quan

  • Thành viên
  • 312 Bài viết

Sau đây là lời giải cho bài 129.

Áp dụng bất đẳng thức Holder, ta có:

$(a^3+b^3+b^3)(b^3+a^3+b^3)(a^3+c^3+b^3)\geq (a^2b+abc+b^3)^3\geq (2ab^2+abc)^3=(ab)^3(2b+c)^3$

Do đó ta có: $\frac{(ab)^3(2b+c)^3}{(a^3+2b^3)^2}\leq a^3+b^3+c^3$. Từ đó ta có$\frac{(2b+c)^3}{c^3(2b^3+a^3)^2}\leq a^3+b^3+c^3$

Tương tự với hai hạng tử còn lại của VT bất đẳng thức từ đó ta thu được đpcm.


:ukliam2:  :ukliam2:  :ukliam2:


#376
an1712

an1712

    Trung sĩ

  • Thành viên
  • 149 Bài viết

Bài 136(IMO Shortlist): Cho a,b,c là các số thực không âm: ab+bc+ca=1. CMR:

$2\leq \frac{1}{2a^2+bc}+\frac{1}{2b^2+ca}+\frac{1}{2c^2+ab}\leq 3(a^2+b^2+c^2)$

áp dụng cauchy có:

$$(\sum \frac{1}{2a^2+bc})^2\leq (\sum (b^2+c^2+bc))(\sum \frac{1}{(b^2+c^2+bc)(2a^2+bc)^2})$$

mà $(b^2+c^2+bc)(2a^2+bc)\geq (ab+bc+ac)^2=1$

bđt $\Leftrightarrow \sum \frac{1}{2a^2+bc}\leq \sum b^2+c^2+bc\leq 3(\sum a^2)$

luôn đúng


Bài viết đã được chỉnh sửa nội dung bởi an1712: 22-06-2015 - 22:30

tiến tới thành công  :D


#377
an1712

an1712

    Trung sĩ

  • Thành viên
  • 149 Bài viết

Sau đây là lời giải cho bài 129.

Áp dụng bất đẳng thức Holder, ta có:

$(a^3+b^3+b^3)(b^3+a^3+b^3)(a^3+c^3+b^3)\geq (a^2b+abc+b^3)^3\geq (2ab^2+abc)^3=(ab)^3(2b+c)^3$

Do đó ta có: $\frac{(ab)^3(2b+c)^3}{(a^3+2b^3)^2}\leq a^3+b^3+c^3$. Từ đó ta có$\frac{(2b+c)^3}{c^3(2b^3+a^3)^2}\leq a^3+b^3+c^3$

Tương tự với hai hạng tử còn lại của VT bất đẳng thức từ đó ta thu được đpcm.

còn cách khác ko?


Bài viết đã được chỉnh sửa nội dung bởi an1712: 22-06-2015 - 16:00

tiến tới thành công  :D


#378
dogsteven

dogsteven

    Đại úy

  • Thành viên
  • 1567 Bài viết

áp dụng cauchy có:

$(\sum \frac{1}{2a^2+bc})^2\leq (\sum (b^2+c^2+bc))(\sum \frac{1}{(b^2+c^2+bc)(2a^2+bc)})$

mà $(b^2+c^2+bc)(2a^2+bc)\geq (ab+bc+ac)^2=1\Leftrightarrow \sum \frac{1}{(b^2+c^2+bc)(2a^2+bc)}\leq 3$

$\sum (bc+b^2+c^2)\leq 3(a^2+b^2+c^2)$

$bđt\Leftrightarrow \sqrt{\sum a^2}\leq \sum a^2$(luôn đúng)

phần cm $\geq 2$ mk thấy hih như sai đề

Phần đầu ta sử dụng CYH: $\sum \dfrac{1}{2a^2+bc}\geqslant \dfrac{4(a+b+c)^2}{\sum (2a^2+bc)(b+c)^2}$

Ta chuyển về chứng minh $p^2+9pr\geqslant 4$. Đồng bậc ta chuyển về chứng minh $-q^2+q+9r\geqslant 0$ với $p=2$

Với $q\leqslant 1$ thì bất đẳng thức hiển nhiên đúng.

Với $8>\dfrac{4}{3}\geqslant q\geqslant 1$ thì $-q^2+q+9r\geqslant -q^2+q+8(q-1)=(8-q)(q-1)\geqslant 0$

Ta có điều phải chứng minh.


Bài viết đã được chỉnh sửa nội dung bởi dogsteven: 22-06-2015 - 16:20

Quyết tâm off dài dài cày hình, số, tổ, rời rạc.


#379
dogsteven

dogsteven

    Đại úy

  • Thành viên
  • 1567 Bài viết

Bài 138(tự sáng tác)(1 bài tập chặt hơn bài 136):  Cho a,b,c >0:ab+ca+bc=1. CMR:

$\frac{1}{a^2+b^2+bc}+\frac{1}{b^2+c^2+ca}+\frac{1}{c^2+a^2+ab}\leq 3(a^2+b^2+c^2)$

$VT=\sum \dfrac{c^2+a^2+bc}{(a^2+b^2+bc)(c^2+a^2+bc)}\leqslant \sum \dfrac{c^2+a^2+bc}{(ab+bc+ca)^2}=2\sum a^2+\sum ab\leqslant 3\sum a^2$


Quyết tâm off dài dài cày hình, số, tổ, rời rạc.


#380
dangkhuong

dangkhuong

    Sĩ quan

  • Thành viên
  • 312 Bài viết

Phần đầu ta sử dụng CYH: $\sum \dfrac{1}{2a^2+bc}\geqslant \dfrac{4(a+b+c)^2}{\sum (2a^2+bc)(b+c)^2}$

Ta chuyển về chứng minh $p^2+6pr\geqslant 4$. Đồng bậc ta chuyển về chứng minh $-q^2+q+9r\geqslant 0$ với $p=2$

Với $q\leqslant 1$ thì bất đẳng thức hiển nhiên đúng.

Với $8>\dfrac{4}{3}\geqslant q\geqslant 1$ thì $-q^2+q+9r\geqslant -q^2+q+8(q-1)=(8-q)(q-1)\geqslant 0$

Ta có điều phải chứng minh.

Dám hỏi dấu bằng xảy ra khi nào???


:ukliam2:  :ukliam2:  :ukliam2:





1 người đang xem chủ đề

0 thành viên, 1 khách, 0 thành viên ẩn danh